LSAT and Law School Admissions Forum

Get expert LSAT preparation and law school admissions advice from PowerScore Test Preparation.

User avatar
 Dave Killoran
PowerScore Staff
  • PowerScore Staff
  • Posts: 5849
  • Joined: Mar 25, 2011
|
#78449
Complete Question Explanation
(The complete setup for this game can be found here: lsat/viewtopic.php?t=15592)

The correct answer choice is (D)

If the computer in office 3 was purchased in 1988, then another chain reaction results. First, from the third rule, the printer in office 4 was purchased in 1988, and from the fourth rule, the computer in office 2 was purchased in 1989. And, because the computer in office 2 was purchased in 1989, we can infer from the first rule that the printer in office 2 was bought in 1989, and we can infer from the second rule that the printer in office 1 was bought in 1989. This leaves the purchase date of the computer in office 4 as the only uncertainty:
PT1-Jun1991 game 2 #12 diagram 1.png
Because this is a Could Be True question, you should expect the answer to trade on an uncertainty in the game, and as the only uncertainty is the purchase date of the computer in office 4, immediately scan the answer choices for mentions of the computer in office 4. Only answer choice (D) addresses the computer in office 4, and as it is possible that the computer in office 4 was purchased in 1987, answer choice (D) is correct.
 sblack1998
  • Posts: 10
  • Joined: Feb 05, 2020
|
#90705
Dave Killoran wrote: Wed Aug 26, 2020 5:46 pm Complete Question Explanation
(The complete setup for this game can be found here: lsat/viewtopic.php?t=15592)

The correct answer choice is (D)

If the computer in office 3 was purchased in 1988, then another chain reaction results. First, from the third rule, the printer in office 4 was purchased in 1988, and from the fourth rule, the computer in office 2 was purchased in 1989. And, because the computer in office 2 was purchased in 1989, we can infer from the first rule that the printer in office 2 was bought in 1989, and we can infer from the second rule that the printer in office 1 was bought in 1989. This leaves the purchase date of the computer in office 4 as the only uncertainty:

PT1-Jun1991 game 2 #12 diagram 1.png

Because this is a Could Be True question, you should expect the answer to trade on an uncertainty in the game, and as the only uncertainty is the purchase date of the computer in office 4, immediately scan the answer choices for mentions of the computer in office 4. Only answer choice (D) addresses the computer in office 4, and as it is possible that the computer in office 4 was purchased in 1987, answer choice (D) is correct.
How did you determine that the printer in office 2 was purchased in 9? Per the rules it could be purchased in 8 or 9.
If the printer in office two was purchased in 8 then the computer in office 2 had to be purchased in 8 as was the printer in office 1. That means the computer in office 4 had to be purchased in 7. I struggled on this question because I had three answer choices that COULD be correct: A, C and D. I eventually chose D based on diagram 2 below because it yielded only one answer. I figured diagram 1 must be wrong in some way. I'm sure it is so obvious that I'm missing it. Here are my diagrams. Where did I go wrong with option 1 diagram?

OPTION 1 ANSWERS A and C and D all fit this answer
P: 8 8/9 8 8
C: 8 8 8 7

OPTION 2 Only answer D works

P: 9 9 8 8
C : 8 9 8 8/7
User avatar
 atierney
PowerScore Staff
  • PowerScore Staff
  • Posts: 215
  • Joined: Jul 06, 2021
|
#90714
Hello,

Remember rule 4 states that the Office #2 computer must be purchased in a different than the Office #3 computer. So, while it could potentially, per other rules, be purchased in 8 or 9, the Local Rule designating #3 in 8 forces it to go in 9. If Office #2 computer is in 9, Office #1 printer must be in 9 per the Rule 2 and cannot be in 8 (A). Note: that's also how we know Office #2 computer generally is only eligible for 8 or 9. Office #2 printer in 8 (C), given Office #2 computer in 9, would violate Rule 1.

D is the only option that could be true here.

Let me know if you have further questions.

Get the most out of your LSAT Prep Plus subscription.

Analyze and track your performance with our Testing and Analytics Package.